3
$\begingroup$

Let $\{v_1, \dotsc, v_m\} \in \mathbb{C}^{2^n}$ be a set of orthonormal vectors. Define a map $R_m$ from $2^n \times 2^n$ to $m \times m$ matrices as follows: $$R_m(M) := \sum_{i,j=1}^m (v_i^*M v_j) E_{ij}$$ where $E_{ij} := e_i e_j^*$ is the all-zeroes matrix with entry 1 at position $(i,j)$. In other words, $R_m$ performs a unitary change of basis and then takes the top-left $3 \times 3$ block of the resulting matrix.

For any $n \geq 2$, show that no matter how the vectors $\{v_i\}$ are chosen, $$\mathrm{U}(3) \not\subseteq \bigl\{ R_3(U_1 \otimes \dotsb \otimes U_n) : U_i \in \mathrm{U}(2) \bigr\}.$$ That is, as matrices $U_i$ range over all possible $2 \times 2$ unitaries, their tensor product will never contain the set of all $3 \times 3$ unitaries in some basis.

More generally, this should hold even if $2 \times 2$ unitaries are replaced by $d \times d$ unitaries and $m = 3$ is replaced by any $m > d$. The motivation for this question comes from "encoded universality" in quantum computing.

Note: I originally asked this question on math.stackexchange but did not receive any answer.

$\endgroup$

1 Answer 1

8
$\begingroup$

When the (general) question is rephrased in less basis-dependent language, I believe that it translates to this: Let $\mathrm{U}(d)$ act on $V = \mathbb{C}^d$ in the usual way, and consider the $n$-fold product $G = \mathrm{U}(d)\times \mathrm{U}(d)\times\cdots\times \mathrm{U}(d)$ acting by the usual tensor product on $$ V^{\otimes n} = \mathbb{C}^d\otimes \mathbb{C}^d\otimes\cdots\otimes \mathbb{C}^d \simeq \mathbb{C}^{d^n}. $$ This is an irreducible $G$-representation, and hence there is, up to a constant multiple, a unique $G$-invariant Hermitian inner product on $V^{\otimes n}$.

Given any subspace $W\subset V^{\otimes n}$, let $\pi_W:V^{\otimes n}\to W$ be the projection onto $W$ that is orthogonal with respect to this $G$-invariant inner product, and define a (smooth) mapping $R_W:G\to \mathrm{End}(W)$ by $$ R_W(g_1,\ldots,g_n)(w) = \pi_W\bigl((g_1\otimes g_2\otimes\cdots\otimes g_n\bigr)(w)\bigr) $$ for $(g_1,\ldots,g_n)\in G$.

The OP asks whether there exists a subspace $W\subset V^{\otimes n}$ of dimension $m>d$ such that $R_W(G)$ contains the group $\mathrm{U}(W)\subset\mathrm{End}(W)$ of unitary transformations of $W$ (when it is endowed with the inner product that it inherits as a subspace of $V^{\otimes n}$). The answer is no, and the following argument provides a proof.

Suppose that such a $W$ existed, and let $H\subset G$ be the (closed) subset $$ H = \left(R_W\right)^{-1}\bigl(\mathrm{U}(W)\bigr). $$ Then I claim that $H$ is a subgroup of $G$, in fact, $H$ is the subgroup of $G$ that preserves the subspace $W\subset V^{\otimes n}$. The reason is that, if $R_W(g_1,\ldots,g_n):W\to W$ is unitary (i.e., preserves lengths), then $\bigl(g_1\otimes g_2\otimes\cdots\otimes g_n\bigr)(w)$ must lie in $W$ for all $w\in W$.

Since $H$ is the compact subgroup of $G$ that preserves the subspace $W$, it follows immediately that $$ R_W:H\to \mathrm{U}(W) $$ is, in fact, a group homomorphism, and a surjective one, at that. Thus, the induced Lie algebra homomorphism $$ r_W:{\frak{h}}\to {\frak{u}}(W) $$ is also surjective. In particular, since ${\frak{u}}(W) = {\frak{z}}\oplus {\frak{su}}(W) \simeq \mathbb{R}\oplus {\frak{su}}(m)$ and ${\frak{su}}(m)$ is simple (since $m>d\ge 2$), there exists an injective Lie algebra homomorphism $\phi: {\frak{su}}(m)\to {\frak h}$ so that the composition $r_W\circ\phi: {\frak{su}}(m) \to {\frak{su}}(W)\subset {\frak{u}}(W)$ is a Lie algebra isomorphism with ${\frak{su}}(W)$.

However, this is not possible, because ${\frak{h}}$ is embedded as a subalgebra $$ {\frak{h}}\subset {\frak{u}}(d)\oplus {\frak{u}}(d)\oplus \cdots \oplus {\frak{u}}(d) = {\frak{g}}, $$ but there can be no nontrivial Lie algebra homomorphism from ${\frak{su}}(m)$ to ${\frak{u}}(d)$ when $m>d$, since ${\frak{su}}(W)$ is simple, and thus, such a homomorphisms would have to be injective, which is impossible for dimension reasons.

$\endgroup$

Your Answer

By clicking “Post Your Answer”, you agree to our terms of service and acknowledge you have read our privacy policy.

Not the answer you're looking for? Browse other questions tagged or ask your own question.